Đến nội dung

Hình ảnh

$ f(f(n)) \le \dfrac{n+f(n)}{2}, \forall n \in \mathbb{N^*}$

- - - - -

  • Please log in to reply
Chủ đề này có 3 trả lời

#1
QUANVU

QUANVU

    B&S-D

  • Hiệp sỹ
  • 4378 Bài viết
Tìm tất cả các song ánh $f:\mathbb{N^*}\rightarrow \mathbb{N^*}$ sao cho:
$$ f(f(n)) \le \dfrac{n+f(n)}{2}, \forall n \in \mathbb{N^*}$$

DDTH

Bài viết đã được chỉnh sửa nội dung bởi E. Galois: 28-08-2012 - 22:43

1728

#2
NGOCTIEN_A1_DQH

NGOCTIEN_A1_DQH

    Never Give Up

  • Thành viên
  • 625 Bài viết

Tìm tất cả các song ánh $f:\mathbb{N^*}\rightarrow \mathbb{N^*}$ sao cho:
$$ f(f(n)) \le \dfrac{n+f(n)}{2}, \forall n \in \mathbb{N^*}$$

DDTH

kí hiệu $ f^k(n) $ là hàm hợp sau khi tác động k lần hàm $ f$

từ giả thiết suy ra $ f(f(n)) \leq \frac{n+f(n)}{2} \leq $ max{$n,f(n)$} $ \forall n \in \mathbb{N^*}$ (*)

nếu tồn tại $ a $ sao cho $ f(a)<a $ thì bằng quy nạp dễ chứng minh được $ f^k(a)<a \forall k \geq 1 $

như vậy tồn tại vô hạn giá trị $ f^k(a)$ trong tập {$1;2;...;a-1$} nên phải tồn tại $ 0<i<j$ sao cho $ f^i(a)=f^j(a)=f^i(f^{j-i}(a))$

mà $ f$ là song ánh nên $ f^{j-i}(a)=a $ (mâu thuẫn vì $ f(a)<a$)

vậy $ f(a) \geq a \forall a>0 $ nên $ f(f(a)) \geq f(a) \forall a>0 $

mặt khác từ (*) ta có: $ f(f(a)) \leq \frac{a+f(a)}{2} \leq f(a) $

$ \Rightarrow f(f(a))=f(a) \Rightarrow f(a)=a$

thử lại thấy thỏa mãn

vậy hàm số cần tìm là hàm $ f(n)=n $

Bài viết đã được chỉnh sửa nội dung bởi NGOCTIEN_A1_DQH: 29-08-2012 - 19:10

Em cắm hoa tươi đặt cạnh bàn

Mong rằng toán học bớt khô khan

Em ơi trong toán nhiều công thức

Cũng đẹp như hoa lại chẳng tàn

#3
tranghieu95

tranghieu95

    Trung sĩ

  • Thành viên
  • 147 Bài viết

Tìm tất cả các song ánh $f:\mathbb{N^*}\rightarrow \mathbb{N^*}$ sao cho:
$$ f(f(n)) \le \dfrac{n+f(n)}{2}, \forall n \in \mathbb{N^*}$$

DDTH

Đặt $f(f(f(....(n))))=f^k(n)$ (n lần k)
Chứng minh bằng quy nạp: $f^k(n) \le \dfrac{2f(n)+n}{3}+\dfrac{3}{2.(-2)^k}.(n-f(n))$(*)
Thâtj vậy:
Dễ thấy (*) đúng với $n=1$
Giả sử (*) đúng với k, ta cần cm (*) đúng với k+1.
Ta có: $f^{k+1}(n) \le \dfrac{f^{k-1}(n)+f^k(n)}{2} \le \dfrac{2.\dfrac{2f(n)+n}{3}+\dfrac{3}{2.(-2)^{k-1}}.(n-f(n))+\dfrac{3}{2.(-2)^k}.(n-f(n))} = \dfrac{2f(n)+n}{3}+\dfrac{3}{2.(-2)^{k+1}}.(n-f(n)$
$\Rightarrow (*)$ đúng
Do đó tồn tại $k_0$ đủ lớn thỏa mãn: $f^{k_0} \le \dfrac{2f(n)+n}{3}+1$
$\Rightarrow \exists k_1 \ne k_2$ tm: $f^{k_1}(n)=f^{k_2}(n)$
$\Rightarrow$ với $\forall n, \exists x_n>1$ tm: $f^{x_n}(n)=n$
Cho $k=x_n$ ta đc: $n \le \dfrac{f(n)+n}{3}+\dfrac{2}{3.(-2)^{x_n}}.(n-f(n)$
$\Rightarrow (f(n)-n).(1-\dfrac{1}{(-2)^{x_n}} \ge 0$
$\Rightarrow f(n) \ge n$
Giả sử $f(n) \ge n \Rightarrow f^{x_n}(x) \ge n$ (MT)
Vậy $f(n)=n$
TỪ TỪ LÀ HẠNH PHÚC
A1K39PBC

#4
PSW

PSW

    Những bài toán trong tuần

  • Quản trị
  • 493 Bài viết
Chấm điểm:
NGOCTIEN_A1_DQH: 10 điểm

tranghieu95: 5 điểm
1) Thể lệ
2) Danh sách các bài toán đã qua: 1-100, 101-200, 201-300, 301-400
Còn chờ gì nữa mà không tham gia! :luoi:




1 người đang xem chủ đề

0 thành viên, 1 khách, 0 thành viên ẩn danh